Last visit was: 19 Nov 2025, 16:24 It is currently 19 Nov 2025, 16:24
Close
GMAT Club Daily Prep
Thank you for using the timer - this advanced tool can estimate your performance and suggest more practice questions. We have subscribed you to Daily Prep Questions via email.

Customized
for You

we will pick new questions that match your level based on your Timer History

Track
Your Progress

every week, we’ll send you an estimated GMAT score based on your performance

Practice
Pays

we will pick new questions that match your level based on your Timer History
Not interested in getting valuable practice questions and articles delivered to your email? No problem, unsubscribe here.
Close
Request Expert Reply
Confirm Cancel
User avatar
Sajjad1994
User avatar
GRE Forum Moderator
Joined: 02 Nov 2016
Last visit: 19 Nov 2025
Posts: 17,304
Own Kudos:
49,313
 [21]
Given Kudos: 6,180
GPA: 3.62
Products:
Posts: 17,304
Kudos: 49,313
 [21]
5
Kudos
Add Kudos
16
Bookmarks
Bookmark this Post
avatar
pradiptirastogi
Joined: 23 Aug 2020
Last visit: 16 Apr 2021
Posts: 81
Own Kudos:
Given Kudos: 20
Posts: 81
Kudos: 7
Kudos
Add Kudos
Bookmarks
Bookmark this Post
User avatar
Basshead
Joined: 09 Jan 2020
Last visit: 07 Feb 2024
Posts: 925
Own Kudos:
Given Kudos: 432
Location: United States
Posts: 925
Kudos: 302
Kudos
Add Kudos
Bookmarks
Bookmark this Post
User avatar
Sajjad1994
User avatar
GRE Forum Moderator
Joined: 02 Nov 2016
Last visit: 19 Nov 2025
Posts: 17,304
Own Kudos:
49,313
 [1]
Given Kudos: 6,180
GPA: 3.62
Products:
Posts: 17,304
Kudos: 49,313
 [1]
1
Kudos
Add Kudos
Bookmarks
Bookmark this Post
pradiptirastogi
Can you please post official explanation for question 2?

Official Explanation

2. It can be inferred that the author

Explanation

Choice (B) is the correct answer because the writer refers to the MetLife study specifically three times and also suggests no other source for facts or claims beyond the MetLife study.

Choice (A) must be ruled out because nothing in the passage suggests this nor is any such claim made.

Choice (C) must be ruled out because nothing in the passage suggests this and there are no facts that could lead to this conclusion.

Choice (D) presents a tempting possibility; this answer constitutes a reasonable conjecture but is, nevertheless, not supported by any fact in the passage.

Choice (E) should be eliminated because the author makes it clear in the first paragraph that baby boomer retirements are at least a “potential threat” if not a code red.

The correct answer is (B).
User avatar
sheldongeorge
Joined: 03 Dec 2015
Last visit: 26 May 2022
Posts: 48
Own Kudos:
Given Kudos: 37
Location: United States
Posts: 48
Kudos: 5
Kudos
Add Kudos
Bookmarks
Bookmark this Post
In Q.1 passage only talks about metlife study 10 years ago ... so (A) - more important than EVER - is too extreme. IMO ans should be (C)
Sajjad1994 pls give OE for Q.1 and Q.3
User avatar
Sajjad1994
User avatar
GRE Forum Moderator
Joined: 02 Nov 2016
Last visit: 19 Nov 2025
Posts: 17,304
Own Kudos:
Given Kudos: 6,180
GPA: 3.62
Products:
Posts: 17,304
Kudos: 49,313
Kudos
Add Kudos
Bookmarks
Bookmark this Post
sheldongeorge
In Q.1 passage only talks about metlife study 10 years ago ... so (A) - more important than EVER - is too extreme. IMO ans should be (C)
Sajjad1994 pls give OE for Q.1 and Q.3

Official Explanation

1. In the passage, the author’s chief concern is to

Difficulty Level: 650

Explanation

Choice (A) presents the focus of the entire first paragraph. Furthermore, all points made in the article have to do with the relationship between benefits and what employees want or value.

Choice (B) is not correct. The author does rely heavily on information from this study to make his or her point; indeed, the study may inform the entire passage, though that is not certain. Nevertheless, the study is used in the service of making a larger point: That benefits may be more important than ever in retaining employees.

Choice (C) is tempting; the author does mention this, but it is the focus of a single paragraph, the last paragraph, and not the entire passage.

Choice (D) is incorrect because the passage never says that. Instead, it makes the point that human resource departments may undervalue the importance of benefits at times when hiring is easier, presumably as a result of high unemployment.

Choice (E) should be eliminated because the author suggests a shifting landscape of benefits; therefore, while benefits remain as important as ever, if not more important, the desired benefits themselves have changed.

The correct answer is (A).

3. Which of the following is NOT mentioned in the passage as a reason for focusing on benefits as a method of employee retention?

Difficulty Level: 750

Explanation

Choice (D) is the only reason not mentioned in the passage.

Choice (A) is mentioned in paragraph 1 as a reason to focus on retention.

Choice (B) is mentioned as a reason to focus on retention in the final paragraph.

Choice (C) is incorrect because the change in employee wants and values is mentioned in paragraph 2, where the author implies that human resource departments must keep up with such change.

Choice (E) should be eliminated because the author does mention this desire and implies that a competent human resources department will keep up with it in order to retain employees.

The correct answer is (D).
User avatar
medha312
Joined: 02 Aug 2018
Last visit: 05 Apr 2023
Posts: 15
Own Kudos:
Given Kudos: 36
Location: India
Posts: 15
Kudos: 6
Kudos
Add Kudos
Bookmarks
Bookmark this Post
please explain Q4. Thanks!
User avatar
Sajjad1994
User avatar
GRE Forum Moderator
Joined: 02 Nov 2016
Last visit: 19 Nov 2025
Posts: 17,304
Own Kudos:
Given Kudos: 6,180
GPA: 3.62
Products:
Posts: 17,304
Kudos: 49,313
Kudos
Add Kudos
Bookmarks
Bookmark this Post
medha312
please explain Q4. Thanks!

Official Explanation

4. With which of the following statements would the author most likely agree?

Difficulty Level: 700

Explanation

At the end of paragraph 2, the author mentions that a keen interest in benefits is now common among younger workers, which suggests this wasn’t the case in the past.

Choice (B) is not correct. The author says in paragraph 2 that the level of employee interest in benefits has burgeoned, that is, increased dramatically.

Choice (C) is incorrect because the author makes it clear that employees appreciate their benefits even when they have to come up with some portion of the cost.

Choice (D) is incorrect because the author states that employees now rate life and disability benefits on a par with opportunity for advancement.

Choice (E) should be eliminated because the author concludes by noting the “increasing value” of voluntary or personalized benefits and their value in creating employee satisfaction and contributing to retention.

The correct answer is (A).
User avatar
VerbalBot
User avatar
Non-Human User
Joined: 01 Oct 2013
Last visit: 04 Jan 2021
Posts: 18,830
Own Kudos:
Posts: 18,830
Kudos: 986
Kudos
Add Kudos
Bookmarks
Bookmark this Post
Hello from the GMAT Club VerbalBot!

Thanks to another GMAT Club member, I have just discovered this valuable topic, yet it had no discussion for over a year. I am now bumping it up - doing my job. I think you may find it valuable (esp those replies with Kudos).

Want to see all other topics I dig out? Follow me (click follow button on profile). You will receive a summary of all topics I bump in your profile area as well as via email.
Moderators:
GMAT Club Verbal Expert
7443 posts
GMAT Club Verbal Expert
231 posts
GRE Forum Moderator
17304 posts
189 posts